LSAT and Law School Admissions Forum

Get expert LSAT preparation and law school admissions advice from PowerScore Test Preparation.

User avatar
 Dave Killoran
PowerScore Staff
  • PowerScore Staff
  • Posts: 5853
  • Joined: Mar 25, 2011
|
#59626
Complete Question Explanation
(The complete setup for this game can be found here: lsat/viewtopic.php?t=15933)

The correct answer choice is (A)

When the L is placed on Tuesday, then from the first rule we know that P is also placed on Tuesday. With Tuesday occupied, both the third and fourth rules are affected: K cannot be placed on Monday (because G cannot be placed on Tuesday), and S cannot be placed on Wednesday (because H cannot be placed on Tuesday). Consequently, K must be placed on Wednesday and S must be placed on Monday. G and H form a dual-option that rotates between Monday and Wednesday:

pt44_o04_g2_q9.png

Answer choice (A): This is the correct answer choice.

Answer choice (B): This answer choice violates the third rule. K cannot be placed on Monday because there is no space for G to be placed on Tuesday.

Answer choice (C): The question stem stipulates that P must be placed on Tuesday, and from the first rule L must also be placed on Tuesday. Therefore, this answer choice cannot occur and it is incorrect.

Answer choice (D): Since L and P occupy the two spaces on Tuesday, no other dog can be placed on Tuesday, and thus this answer choice is incorrect.

Answer choice (E): This answer choice violates the fourth rule. S cannot be placed on Wednesday because there is no space for H to be placed on Tuesday.
You do not have the required permissions to view the files attached to this post.

Get the most out of your LSAT Prep Plus subscription.

Analyze and track your performance with our Testing and Analytics Package.